Scientist: To study the comparative effectiveness of two experimental medications for athlete's foot, a representativ...

Cynthia-Lee on September 10, 2018

Please explain this question,thanks

I thought the flaw that the reporter made is that just because only people whose athlete's foot was cured are the ones receiving medication M doesn't mean ones whose athlete's foot not being cured doesn't receive medication M. It's it possible that people in the study who received medication M but their athlete's foot are not being cured. But I don't see why A is correct. Also why can't E be right. Please help to explain this question, Thank you.

Reply
Create a free account to read and take part in forum discussions.

Already have an account? log in

Max-Youngquist on September 14, 2018

@cynthia-lee it helps to see (A) if we diagram:

Not Athletes Foot (not AF) ==> Medication M (M)
not M ==> AF

Note that we cannot conclude anything from AF, only not AF. In order for anything to be true for all AF, it would have to be a sufficient condition. But that is not the case. Basically the error in reasoning the author made can be diagrammed:

AF ==> not M
M ==> not AF

So while the specific wording in the conclusion made the error (AF ==> not M), the contrapositive is THE SAME ARGUMENT: M ==> not AF. That is what answer choice (A) lists.

(E) is wrong because the author's flaw is based on making the error (AF ==> not M, M ==> not AF), but answer choice E is talking about not M ==> not AF.